$frac{|a|}{|b-c|} + frac{|b|}{|c-a|} + frac{|c|}{|b-a|} geq 2$











up vote
5
down vote

favorite












If $a, b, c$ are distinct real numbers then you demonstrate that:



$$ S=frac{|a|}{|b-c|} + frac{|b|}{|c-a|} + frac{|c|}{|b-a|} geq 2.$$



Using inequality $ |x-y|leq |x|+|y|$ we showed that $ S >frac{2}{3}.$



For $b = 2a, c = 3a, S=5,$ that is, the sum has values greater than 2, but I have not been able to prove this.










share|cite|improve this question






















  • Put everyting on one side, multiply everything by $|b-c||c-a||b-a|, and go from there.
    – user3482749
    yesterday















up vote
5
down vote

favorite












If $a, b, c$ are distinct real numbers then you demonstrate that:



$$ S=frac{|a|}{|b-c|} + frac{|b|}{|c-a|} + frac{|c|}{|b-a|} geq 2.$$



Using inequality $ |x-y|leq |x|+|y|$ we showed that $ S >frac{2}{3}.$



For $b = 2a, c = 3a, S=5,$ that is, the sum has values greater than 2, but I have not been able to prove this.










share|cite|improve this question






















  • Put everyting on one side, multiply everything by $|b-c||c-a||b-a|, and go from there.
    – user3482749
    yesterday













up vote
5
down vote

favorite









up vote
5
down vote

favorite











If $a, b, c$ are distinct real numbers then you demonstrate that:



$$ S=frac{|a|}{|b-c|} + frac{|b|}{|c-a|} + frac{|c|}{|b-a|} geq 2.$$



Using inequality $ |x-y|leq |x|+|y|$ we showed that $ S >frac{2}{3}.$



For $b = 2a, c = 3a, S=5,$ that is, the sum has values greater than 2, but I have not been able to prove this.










share|cite|improve this question













If $a, b, c$ are distinct real numbers then you demonstrate that:



$$ S=frac{|a|}{|b-c|} + frac{|b|}{|c-a|} + frac{|c|}{|b-a|} geq 2.$$



Using inequality $ |x-y|leq |x|+|y|$ we showed that $ S >frac{2}{3}.$



For $b = 2a, c = 3a, S=5,$ that is, the sum has values greater than 2, but I have not been able to prove this.







inequality






share|cite|improve this question













share|cite|improve this question











share|cite|improve this question




share|cite|improve this question










asked yesterday









medicu

3,016813




3,016813












  • Put everyting on one side, multiply everything by $|b-c||c-a||b-a|, and go from there.
    – user3482749
    yesterday


















  • Put everyting on one side, multiply everything by $|b-c||c-a||b-a|, and go from there.
    – user3482749
    yesterday
















Put everyting on one side, multiply everything by $|b-c||c-a||b-a|, and go from there.
– user3482749
yesterday




Put everyting on one side, multiply everything by $|b-c||c-a||b-a|, and go from there.
– user3482749
yesterday










4 Answers
4






active

oldest

votes

















up vote
4
down vote



accepted










Let $frac{a}{b-c}=x$, $frac{b}{c-a}=y$ and $frac{c}{a-b}=z$.



Thus, $$xy+xz+yz=sum_{cyc}frac{ab}{(b-c)(c-a)}=frac{sumlimits_{cyc}ab(a-b)}{(a-b)(b-c)(c-a)}=frac{(a-b)(a-c)(b-c)}{(a-b)(b-c)(c-a)}=-1.$$
Id est,
$$sum_{cyc}left|frac{a}{b-c}right|=sqrt{left(|x|+|y|+|z|right)^2}=sqrt{x^2+y^2+z^2+2sumlimits_{cyc}|xy|}=$$
$$=sqrt{(x+y+z)^2+2+2sumlimits_{cyc}|xy|}geqsqrt{2+2}=2.$$






share|cite|improve this answer

















  • 1




    +1, nice answer.
    – the_candyman
    yesterday


















up vote
3
down vote













Without loss of generality we can assume that $b$ and $c$ have the same sign. Then $|b|=|b-c|+|c|$. Also, $|c-a|le |c|+|a|$ and $|a-b| le |a|+|b-c|+|c|$. Therefore
$$
frac{|a|}{|b-c|}+frac{|b|}{|c-a|}+frac{|c|}{|a-b|} ge frac{|a|}{|b-c|} + frac{|b-c|+|c|}{|c|+|a|} + frac{|c|}{|a|+|b-c|+|c|} = (*).$$



Denote $x=|a|$, $y=|b-c|$, $z=|c|$. Then, by Schwarz and by $x^2+y^2 ge 2xy$, we get
begin{align*}
(*) & = frac{x}{y}+frac{y+z}{z+x}+frac{z}{x+y+z} \
&ge frac{(x+(y+z)+z)^2}{xy+(y+z)(z+x)+z(x+y+z)} \
&= frac{x^2+y^2+4z^2+2xy+4yz+4zx}{2z^2+2xy+2yz+2zx} \
&ge frac{2xy+4z^2+2xy+4yz+4zx}{2z^2+2xy+2yz+2zx} \
&=2.end{align*}






share|cite|improve this answer





















  • +1, nice answer.
    – the_candyman
    yesterday


















up vote
0
down vote













Notice that $|x-y|leq |x|+|y|$ implies that:
$$frac{|z|}{|x-y|} geq frac{|z|}{|x|+|y|}.$$
Then:



$$S geq frac{|a|}{|b|+|c|} + frac{|b|}{|a|+|c|} + frac{|c|}{|a|+|b|} = \
=frac{A}{B+C} + frac{B}{A+C} + frac{C}{A+B},$$

where I set $A = |a|, B = |b|$ and $C= |c|$ for the sake of simplicity.



Multiplying and dividing numerators and denominators by $(B+C)(A+C)(A+B)$, we get that:



$$S geq frac{A^3+A^2B + A^2 C + AB^2 + 3ABC + AC^2 + B^3 + B^2C + BC^2 + C^3 }{(B+C)(A+C)(A+B)}.$$



Notice that the denominator can be expanded as follows:



$$D = (B+C)(A+C)(A+B) = A^2B + A^2C + AB^2 + 2ABC + AC^2 + B^2C + BC^2.$$



Therefore, the numerator can be rewritten as:



$$ N= D + A^3 +B^3 + C^3 + ABC.$$



In other words:



$$S geq frac{D + A^3 +B^3 + C^3 + ABC}{D} > frac{D}{D} = 1.$$






share|cite|improve this answer























  • How does this prove that $S ge 2$?
    – timon92
    yesterday










  • @timon92 It doesn't. This is my attempt. If I can find a better lower bound, then I will post it.
    – the_candyman
    yesterday






  • 1




    Fair enough. Also, using the so-called Nesbitt inequality one can get better bound: $Sge frac{|a|}{|b|+|c|}+frac{|b|}{|c|+|a|}+frac{|c|}{|a|+|b|} ge frac 32.$
    – timon92
    yesterday


















up vote
0
down vote













I tried to give a simple proof by reducing to simpler and simpler cases, all the time doing "nothing", and the final rephrasing is simple, too.




  • First of all, let us note that the (LHS of the) given inequality is invariated by permutations of the letters / variables $a,b,c$. So we may and do assume $ale ble c$.


  • Let $s,t>0$ be then such that $b=a+s$, $c=b+t=a+s+t$.


  • Replacing the initial $a,b,c$ with $-c,-b,-a$, we may and do assume $bge 0$.


  • The given inequality can now be rewritten equivalently:
    $$
    S=
    frac{|a|}t +
    frac{|a+s|}{s+t} +
    frac{|a+s+t|}s
    ge 2 .
    $$


  • If $a$ is $ge0$, then all three numbers $a,b,c$ are $ge 0$, and we can write
    $$
    begin{aligned}
    S
    &= frac{|a|}t +
    frac{|a+s|}{s+t} +
    frac{|a+s+t|}s
    \
    &ge
    frac{0}t +
    frac{s}{s+t} +
    frac{s+t}s
    %ge 2sqrt{
    %frac{s}{s+t} cdot
    %frac{s+t}s}
    ge2 ,qquad(age 0)
    end{aligned}
    $$

    and the last $ge$ (AM-GM) is moreover $>0$ (because $tne 0$). The case with $a < b < c=a+s+tle 0$ is similar.


  • So the single interesting case is the one with $a<0le b<c$ . We rewrite the inequality in the form:
    $$
    frac{|b-s|}t+frac b{s+t}+frac{b+t}sge 2 .
    $$

    Here, $b$ is allowed to vary between $0$ and $s$, and the first modulus is $|a|=-a=-(b-s)=s-b$, so we have to show:
    $$
    frac{s-b}t+frac b{s+t}+frac{b+t}sge 2 .
    $$

    But this is a linear (ergo convex) function in $b$, so the inequality has to be tested only for the possible extremities, $b=0$ and $b=s$. For $b=0$ we obtain $frac st+frac tsge 2$, true, with equality for $s=t$. For $b=s$ we get $frac s{s+t}+frac {s+t}sge 2$, an inequality of the same shape, but we cannot get equality. (For intermediate values of $b$ we get intermediate values of the expression.)







share|cite|improve this answer





















    Your Answer





    StackExchange.ifUsing("editor", function () {
    return StackExchange.using("mathjaxEditing", function () {
    StackExchange.MarkdownEditor.creationCallbacks.add(function (editor, postfix) {
    StackExchange.mathjaxEditing.prepareWmdForMathJax(editor, postfix, [["$", "$"], ["\\(","\\)"]]);
    });
    });
    }, "mathjax-editing");

    StackExchange.ready(function() {
    var channelOptions = {
    tags: "".split(" "),
    id: "69"
    };
    initTagRenderer("".split(" "), "".split(" "), channelOptions);

    StackExchange.using("externalEditor", function() {
    // Have to fire editor after snippets, if snippets enabled
    if (StackExchange.settings.snippets.snippetsEnabled) {
    StackExchange.using("snippets", function() {
    createEditor();
    });
    }
    else {
    createEditor();
    }
    });

    function createEditor() {
    StackExchange.prepareEditor({
    heartbeatType: 'answer',
    convertImagesToLinks: true,
    noModals: true,
    showLowRepImageUploadWarning: true,
    reputationToPostImages: 10,
    bindNavPrevention: true,
    postfix: "",
    imageUploader: {
    brandingHtml: "Powered by u003ca class="icon-imgur-white" href="https://imgur.com/"u003eu003c/au003e",
    contentPolicyHtml: "User contributions licensed under u003ca href="https://creativecommons.org/licenses/by-sa/3.0/"u003ecc by-sa 3.0 with attribution requiredu003c/au003e u003ca href="https://stackoverflow.com/legal/content-policy"u003e(content policy)u003c/au003e",
    allowUrls: true
    },
    noCode: true, onDemand: true,
    discardSelector: ".discard-answer"
    ,immediatelyShowMarkdownHelp:true
    });


    }
    });














     

    draft saved


    draft discarded


















    StackExchange.ready(
    function () {
    StackExchange.openid.initPostLogin('.new-post-login', 'https%3a%2f%2fmath.stackexchange.com%2fquestions%2f3005207%2ffracab-c-fracbc-a-fraccb-a-geq-2%23new-answer', 'question_page');
    }
    );

    Post as a guest















    Required, but never shown

























    4 Answers
    4






    active

    oldest

    votes








    4 Answers
    4






    active

    oldest

    votes









    active

    oldest

    votes






    active

    oldest

    votes








    up vote
    4
    down vote



    accepted










    Let $frac{a}{b-c}=x$, $frac{b}{c-a}=y$ and $frac{c}{a-b}=z$.



    Thus, $$xy+xz+yz=sum_{cyc}frac{ab}{(b-c)(c-a)}=frac{sumlimits_{cyc}ab(a-b)}{(a-b)(b-c)(c-a)}=frac{(a-b)(a-c)(b-c)}{(a-b)(b-c)(c-a)}=-1.$$
    Id est,
    $$sum_{cyc}left|frac{a}{b-c}right|=sqrt{left(|x|+|y|+|z|right)^2}=sqrt{x^2+y^2+z^2+2sumlimits_{cyc}|xy|}=$$
    $$=sqrt{(x+y+z)^2+2+2sumlimits_{cyc}|xy|}geqsqrt{2+2}=2.$$






    share|cite|improve this answer

















    • 1




      +1, nice answer.
      – the_candyman
      yesterday















    up vote
    4
    down vote



    accepted










    Let $frac{a}{b-c}=x$, $frac{b}{c-a}=y$ and $frac{c}{a-b}=z$.



    Thus, $$xy+xz+yz=sum_{cyc}frac{ab}{(b-c)(c-a)}=frac{sumlimits_{cyc}ab(a-b)}{(a-b)(b-c)(c-a)}=frac{(a-b)(a-c)(b-c)}{(a-b)(b-c)(c-a)}=-1.$$
    Id est,
    $$sum_{cyc}left|frac{a}{b-c}right|=sqrt{left(|x|+|y|+|z|right)^2}=sqrt{x^2+y^2+z^2+2sumlimits_{cyc}|xy|}=$$
    $$=sqrt{(x+y+z)^2+2+2sumlimits_{cyc}|xy|}geqsqrt{2+2}=2.$$






    share|cite|improve this answer

















    • 1




      +1, nice answer.
      – the_candyman
      yesterday













    up vote
    4
    down vote



    accepted







    up vote
    4
    down vote



    accepted






    Let $frac{a}{b-c}=x$, $frac{b}{c-a}=y$ and $frac{c}{a-b}=z$.



    Thus, $$xy+xz+yz=sum_{cyc}frac{ab}{(b-c)(c-a)}=frac{sumlimits_{cyc}ab(a-b)}{(a-b)(b-c)(c-a)}=frac{(a-b)(a-c)(b-c)}{(a-b)(b-c)(c-a)}=-1.$$
    Id est,
    $$sum_{cyc}left|frac{a}{b-c}right|=sqrt{left(|x|+|y|+|z|right)^2}=sqrt{x^2+y^2+z^2+2sumlimits_{cyc}|xy|}=$$
    $$=sqrt{(x+y+z)^2+2+2sumlimits_{cyc}|xy|}geqsqrt{2+2}=2.$$






    share|cite|improve this answer












    Let $frac{a}{b-c}=x$, $frac{b}{c-a}=y$ and $frac{c}{a-b}=z$.



    Thus, $$xy+xz+yz=sum_{cyc}frac{ab}{(b-c)(c-a)}=frac{sumlimits_{cyc}ab(a-b)}{(a-b)(b-c)(c-a)}=frac{(a-b)(a-c)(b-c)}{(a-b)(b-c)(c-a)}=-1.$$
    Id est,
    $$sum_{cyc}left|frac{a}{b-c}right|=sqrt{left(|x|+|y|+|z|right)^2}=sqrt{x^2+y^2+z^2+2sumlimits_{cyc}|xy|}=$$
    $$=sqrt{(x+y+z)^2+2+2sumlimits_{cyc}|xy|}geqsqrt{2+2}=2.$$







    share|cite|improve this answer












    share|cite|improve this answer



    share|cite|improve this answer










    answered yesterday









    Michael Rozenberg

    94.2k1588183




    94.2k1588183








    • 1




      +1, nice answer.
      – the_candyman
      yesterday














    • 1




      +1, nice answer.
      – the_candyman
      yesterday








    1




    1




    +1, nice answer.
    – the_candyman
    yesterday




    +1, nice answer.
    – the_candyman
    yesterday










    up vote
    3
    down vote













    Without loss of generality we can assume that $b$ and $c$ have the same sign. Then $|b|=|b-c|+|c|$. Also, $|c-a|le |c|+|a|$ and $|a-b| le |a|+|b-c|+|c|$. Therefore
    $$
    frac{|a|}{|b-c|}+frac{|b|}{|c-a|}+frac{|c|}{|a-b|} ge frac{|a|}{|b-c|} + frac{|b-c|+|c|}{|c|+|a|} + frac{|c|}{|a|+|b-c|+|c|} = (*).$$



    Denote $x=|a|$, $y=|b-c|$, $z=|c|$. Then, by Schwarz and by $x^2+y^2 ge 2xy$, we get
    begin{align*}
    (*) & = frac{x}{y}+frac{y+z}{z+x}+frac{z}{x+y+z} \
    &ge frac{(x+(y+z)+z)^2}{xy+(y+z)(z+x)+z(x+y+z)} \
    &= frac{x^2+y^2+4z^2+2xy+4yz+4zx}{2z^2+2xy+2yz+2zx} \
    &ge frac{2xy+4z^2+2xy+4yz+4zx}{2z^2+2xy+2yz+2zx} \
    &=2.end{align*}






    share|cite|improve this answer





















    • +1, nice answer.
      – the_candyman
      yesterday















    up vote
    3
    down vote













    Without loss of generality we can assume that $b$ and $c$ have the same sign. Then $|b|=|b-c|+|c|$. Also, $|c-a|le |c|+|a|$ and $|a-b| le |a|+|b-c|+|c|$. Therefore
    $$
    frac{|a|}{|b-c|}+frac{|b|}{|c-a|}+frac{|c|}{|a-b|} ge frac{|a|}{|b-c|} + frac{|b-c|+|c|}{|c|+|a|} + frac{|c|}{|a|+|b-c|+|c|} = (*).$$



    Denote $x=|a|$, $y=|b-c|$, $z=|c|$. Then, by Schwarz and by $x^2+y^2 ge 2xy$, we get
    begin{align*}
    (*) & = frac{x}{y}+frac{y+z}{z+x}+frac{z}{x+y+z} \
    &ge frac{(x+(y+z)+z)^2}{xy+(y+z)(z+x)+z(x+y+z)} \
    &= frac{x^2+y^2+4z^2+2xy+4yz+4zx}{2z^2+2xy+2yz+2zx} \
    &ge frac{2xy+4z^2+2xy+4yz+4zx}{2z^2+2xy+2yz+2zx} \
    &=2.end{align*}






    share|cite|improve this answer





















    • +1, nice answer.
      – the_candyman
      yesterday













    up vote
    3
    down vote










    up vote
    3
    down vote









    Without loss of generality we can assume that $b$ and $c$ have the same sign. Then $|b|=|b-c|+|c|$. Also, $|c-a|le |c|+|a|$ and $|a-b| le |a|+|b-c|+|c|$. Therefore
    $$
    frac{|a|}{|b-c|}+frac{|b|}{|c-a|}+frac{|c|}{|a-b|} ge frac{|a|}{|b-c|} + frac{|b-c|+|c|}{|c|+|a|} + frac{|c|}{|a|+|b-c|+|c|} = (*).$$



    Denote $x=|a|$, $y=|b-c|$, $z=|c|$. Then, by Schwarz and by $x^2+y^2 ge 2xy$, we get
    begin{align*}
    (*) & = frac{x}{y}+frac{y+z}{z+x}+frac{z}{x+y+z} \
    &ge frac{(x+(y+z)+z)^2}{xy+(y+z)(z+x)+z(x+y+z)} \
    &= frac{x^2+y^2+4z^2+2xy+4yz+4zx}{2z^2+2xy+2yz+2zx} \
    &ge frac{2xy+4z^2+2xy+4yz+4zx}{2z^2+2xy+2yz+2zx} \
    &=2.end{align*}






    share|cite|improve this answer












    Without loss of generality we can assume that $b$ and $c$ have the same sign. Then $|b|=|b-c|+|c|$. Also, $|c-a|le |c|+|a|$ and $|a-b| le |a|+|b-c|+|c|$. Therefore
    $$
    frac{|a|}{|b-c|}+frac{|b|}{|c-a|}+frac{|c|}{|a-b|} ge frac{|a|}{|b-c|} + frac{|b-c|+|c|}{|c|+|a|} + frac{|c|}{|a|+|b-c|+|c|} = (*).$$



    Denote $x=|a|$, $y=|b-c|$, $z=|c|$. Then, by Schwarz and by $x^2+y^2 ge 2xy$, we get
    begin{align*}
    (*) & = frac{x}{y}+frac{y+z}{z+x}+frac{z}{x+y+z} \
    &ge frac{(x+(y+z)+z)^2}{xy+(y+z)(z+x)+z(x+y+z)} \
    &= frac{x^2+y^2+4z^2+2xy+4yz+4zx}{2z^2+2xy+2yz+2zx} \
    &ge frac{2xy+4z^2+2xy+4yz+4zx}{2z^2+2xy+2yz+2zx} \
    &=2.end{align*}







    share|cite|improve this answer












    share|cite|improve this answer



    share|cite|improve this answer










    answered yesterday









    timon92

    4,0691725




    4,0691725












    • +1, nice answer.
      – the_candyman
      yesterday


















    • +1, nice answer.
      – the_candyman
      yesterday
















    +1, nice answer.
    – the_candyman
    yesterday




    +1, nice answer.
    – the_candyman
    yesterday










    up vote
    0
    down vote













    Notice that $|x-y|leq |x|+|y|$ implies that:
    $$frac{|z|}{|x-y|} geq frac{|z|}{|x|+|y|}.$$
    Then:



    $$S geq frac{|a|}{|b|+|c|} + frac{|b|}{|a|+|c|} + frac{|c|}{|a|+|b|} = \
    =frac{A}{B+C} + frac{B}{A+C} + frac{C}{A+B},$$

    where I set $A = |a|, B = |b|$ and $C= |c|$ for the sake of simplicity.



    Multiplying and dividing numerators and denominators by $(B+C)(A+C)(A+B)$, we get that:



    $$S geq frac{A^3+A^2B + A^2 C + AB^2 + 3ABC + AC^2 + B^3 + B^2C + BC^2 + C^3 }{(B+C)(A+C)(A+B)}.$$



    Notice that the denominator can be expanded as follows:



    $$D = (B+C)(A+C)(A+B) = A^2B + A^2C + AB^2 + 2ABC + AC^2 + B^2C + BC^2.$$



    Therefore, the numerator can be rewritten as:



    $$ N= D + A^3 +B^3 + C^3 + ABC.$$



    In other words:



    $$S geq frac{D + A^3 +B^3 + C^3 + ABC}{D} > frac{D}{D} = 1.$$






    share|cite|improve this answer























    • How does this prove that $S ge 2$?
      – timon92
      yesterday










    • @timon92 It doesn't. This is my attempt. If I can find a better lower bound, then I will post it.
      – the_candyman
      yesterday






    • 1




      Fair enough. Also, using the so-called Nesbitt inequality one can get better bound: $Sge frac{|a|}{|b|+|c|}+frac{|b|}{|c|+|a|}+frac{|c|}{|a|+|b|} ge frac 32.$
      – timon92
      yesterday















    up vote
    0
    down vote













    Notice that $|x-y|leq |x|+|y|$ implies that:
    $$frac{|z|}{|x-y|} geq frac{|z|}{|x|+|y|}.$$
    Then:



    $$S geq frac{|a|}{|b|+|c|} + frac{|b|}{|a|+|c|} + frac{|c|}{|a|+|b|} = \
    =frac{A}{B+C} + frac{B}{A+C} + frac{C}{A+B},$$

    where I set $A = |a|, B = |b|$ and $C= |c|$ for the sake of simplicity.



    Multiplying and dividing numerators and denominators by $(B+C)(A+C)(A+B)$, we get that:



    $$S geq frac{A^3+A^2B + A^2 C + AB^2 + 3ABC + AC^2 + B^3 + B^2C + BC^2 + C^3 }{(B+C)(A+C)(A+B)}.$$



    Notice that the denominator can be expanded as follows:



    $$D = (B+C)(A+C)(A+B) = A^2B + A^2C + AB^2 + 2ABC + AC^2 + B^2C + BC^2.$$



    Therefore, the numerator can be rewritten as:



    $$ N= D + A^3 +B^3 + C^3 + ABC.$$



    In other words:



    $$S geq frac{D + A^3 +B^3 + C^3 + ABC}{D} > frac{D}{D} = 1.$$






    share|cite|improve this answer























    • How does this prove that $S ge 2$?
      – timon92
      yesterday










    • @timon92 It doesn't. This is my attempt. If I can find a better lower bound, then I will post it.
      – the_candyman
      yesterday






    • 1




      Fair enough. Also, using the so-called Nesbitt inequality one can get better bound: $Sge frac{|a|}{|b|+|c|}+frac{|b|}{|c|+|a|}+frac{|c|}{|a|+|b|} ge frac 32.$
      – timon92
      yesterday













    up vote
    0
    down vote










    up vote
    0
    down vote









    Notice that $|x-y|leq |x|+|y|$ implies that:
    $$frac{|z|}{|x-y|} geq frac{|z|}{|x|+|y|}.$$
    Then:



    $$S geq frac{|a|}{|b|+|c|} + frac{|b|}{|a|+|c|} + frac{|c|}{|a|+|b|} = \
    =frac{A}{B+C} + frac{B}{A+C} + frac{C}{A+B},$$

    where I set $A = |a|, B = |b|$ and $C= |c|$ for the sake of simplicity.



    Multiplying and dividing numerators and denominators by $(B+C)(A+C)(A+B)$, we get that:



    $$S geq frac{A^3+A^2B + A^2 C + AB^2 + 3ABC + AC^2 + B^3 + B^2C + BC^2 + C^3 }{(B+C)(A+C)(A+B)}.$$



    Notice that the denominator can be expanded as follows:



    $$D = (B+C)(A+C)(A+B) = A^2B + A^2C + AB^2 + 2ABC + AC^2 + B^2C + BC^2.$$



    Therefore, the numerator can be rewritten as:



    $$ N= D + A^3 +B^3 + C^3 + ABC.$$



    In other words:



    $$S geq frac{D + A^3 +B^3 + C^3 + ABC}{D} > frac{D}{D} = 1.$$






    share|cite|improve this answer














    Notice that $|x-y|leq |x|+|y|$ implies that:
    $$frac{|z|}{|x-y|} geq frac{|z|}{|x|+|y|}.$$
    Then:



    $$S geq frac{|a|}{|b|+|c|} + frac{|b|}{|a|+|c|} + frac{|c|}{|a|+|b|} = \
    =frac{A}{B+C} + frac{B}{A+C} + frac{C}{A+B},$$

    where I set $A = |a|, B = |b|$ and $C= |c|$ for the sake of simplicity.



    Multiplying and dividing numerators and denominators by $(B+C)(A+C)(A+B)$, we get that:



    $$S geq frac{A^3+A^2B + A^2 C + AB^2 + 3ABC + AC^2 + B^3 + B^2C + BC^2 + C^3 }{(B+C)(A+C)(A+B)}.$$



    Notice that the denominator can be expanded as follows:



    $$D = (B+C)(A+C)(A+B) = A^2B + A^2C + AB^2 + 2ABC + AC^2 + B^2C + BC^2.$$



    Therefore, the numerator can be rewritten as:



    $$ N= D + A^3 +B^3 + C^3 + ABC.$$



    In other words:



    $$S geq frac{D + A^3 +B^3 + C^3 + ABC}{D} > frac{D}{D} = 1.$$







    share|cite|improve this answer














    share|cite|improve this answer



    share|cite|improve this answer








    edited yesterday

























    answered yesterday









    the_candyman

    8,54921944




    8,54921944












    • How does this prove that $S ge 2$?
      – timon92
      yesterday










    • @timon92 It doesn't. This is my attempt. If I can find a better lower bound, then I will post it.
      – the_candyman
      yesterday






    • 1




      Fair enough. Also, using the so-called Nesbitt inequality one can get better bound: $Sge frac{|a|}{|b|+|c|}+frac{|b|}{|c|+|a|}+frac{|c|}{|a|+|b|} ge frac 32.$
      – timon92
      yesterday


















    • How does this prove that $S ge 2$?
      – timon92
      yesterday










    • @timon92 It doesn't. This is my attempt. If I can find a better lower bound, then I will post it.
      – the_candyman
      yesterday






    • 1




      Fair enough. Also, using the so-called Nesbitt inequality one can get better bound: $Sge frac{|a|}{|b|+|c|}+frac{|b|}{|c|+|a|}+frac{|c|}{|a|+|b|} ge frac 32.$
      – timon92
      yesterday
















    How does this prove that $S ge 2$?
    – timon92
    yesterday




    How does this prove that $S ge 2$?
    – timon92
    yesterday












    @timon92 It doesn't. This is my attempt. If I can find a better lower bound, then I will post it.
    – the_candyman
    yesterday




    @timon92 It doesn't. This is my attempt. If I can find a better lower bound, then I will post it.
    – the_candyman
    yesterday




    1




    1




    Fair enough. Also, using the so-called Nesbitt inequality one can get better bound: $Sge frac{|a|}{|b|+|c|}+frac{|b|}{|c|+|a|}+frac{|c|}{|a|+|b|} ge frac 32.$
    – timon92
    yesterday




    Fair enough. Also, using the so-called Nesbitt inequality one can get better bound: $Sge frac{|a|}{|b|+|c|}+frac{|b|}{|c|+|a|}+frac{|c|}{|a|+|b|} ge frac 32.$
    – timon92
    yesterday










    up vote
    0
    down vote













    I tried to give a simple proof by reducing to simpler and simpler cases, all the time doing "nothing", and the final rephrasing is simple, too.




    • First of all, let us note that the (LHS of the) given inequality is invariated by permutations of the letters / variables $a,b,c$. So we may and do assume $ale ble c$.


    • Let $s,t>0$ be then such that $b=a+s$, $c=b+t=a+s+t$.


    • Replacing the initial $a,b,c$ with $-c,-b,-a$, we may and do assume $bge 0$.


    • The given inequality can now be rewritten equivalently:
      $$
      S=
      frac{|a|}t +
      frac{|a+s|}{s+t} +
      frac{|a+s+t|}s
      ge 2 .
      $$


    • If $a$ is $ge0$, then all three numbers $a,b,c$ are $ge 0$, and we can write
      $$
      begin{aligned}
      S
      &= frac{|a|}t +
      frac{|a+s|}{s+t} +
      frac{|a+s+t|}s
      \
      &ge
      frac{0}t +
      frac{s}{s+t} +
      frac{s+t}s
      %ge 2sqrt{
      %frac{s}{s+t} cdot
      %frac{s+t}s}
      ge2 ,qquad(age 0)
      end{aligned}
      $$

      and the last $ge$ (AM-GM) is moreover $>0$ (because $tne 0$). The case with $a < b < c=a+s+tle 0$ is similar.


    • So the single interesting case is the one with $a<0le b<c$ . We rewrite the inequality in the form:
      $$
      frac{|b-s|}t+frac b{s+t}+frac{b+t}sge 2 .
      $$

      Here, $b$ is allowed to vary between $0$ and $s$, and the first modulus is $|a|=-a=-(b-s)=s-b$, so we have to show:
      $$
      frac{s-b}t+frac b{s+t}+frac{b+t}sge 2 .
      $$

      But this is a linear (ergo convex) function in $b$, so the inequality has to be tested only for the possible extremities, $b=0$ and $b=s$. For $b=0$ we obtain $frac st+frac tsge 2$, true, with equality for $s=t$. For $b=s$ we get $frac s{s+t}+frac {s+t}sge 2$, an inequality of the same shape, but we cannot get equality. (For intermediate values of $b$ we get intermediate values of the expression.)







    share|cite|improve this answer

























      up vote
      0
      down vote













      I tried to give a simple proof by reducing to simpler and simpler cases, all the time doing "nothing", and the final rephrasing is simple, too.




      • First of all, let us note that the (LHS of the) given inequality is invariated by permutations of the letters / variables $a,b,c$. So we may and do assume $ale ble c$.


      • Let $s,t>0$ be then such that $b=a+s$, $c=b+t=a+s+t$.


      • Replacing the initial $a,b,c$ with $-c,-b,-a$, we may and do assume $bge 0$.


      • The given inequality can now be rewritten equivalently:
        $$
        S=
        frac{|a|}t +
        frac{|a+s|}{s+t} +
        frac{|a+s+t|}s
        ge 2 .
        $$


      • If $a$ is $ge0$, then all three numbers $a,b,c$ are $ge 0$, and we can write
        $$
        begin{aligned}
        S
        &= frac{|a|}t +
        frac{|a+s|}{s+t} +
        frac{|a+s+t|}s
        \
        &ge
        frac{0}t +
        frac{s}{s+t} +
        frac{s+t}s
        %ge 2sqrt{
        %frac{s}{s+t} cdot
        %frac{s+t}s}
        ge2 ,qquad(age 0)
        end{aligned}
        $$

        and the last $ge$ (AM-GM) is moreover $>0$ (because $tne 0$). The case with $a < b < c=a+s+tle 0$ is similar.


      • So the single interesting case is the one with $a<0le b<c$ . We rewrite the inequality in the form:
        $$
        frac{|b-s|}t+frac b{s+t}+frac{b+t}sge 2 .
        $$

        Here, $b$ is allowed to vary between $0$ and $s$, and the first modulus is $|a|=-a=-(b-s)=s-b$, so we have to show:
        $$
        frac{s-b}t+frac b{s+t}+frac{b+t}sge 2 .
        $$

        But this is a linear (ergo convex) function in $b$, so the inequality has to be tested only for the possible extremities, $b=0$ and $b=s$. For $b=0$ we obtain $frac st+frac tsge 2$, true, with equality for $s=t$. For $b=s$ we get $frac s{s+t}+frac {s+t}sge 2$, an inequality of the same shape, but we cannot get equality. (For intermediate values of $b$ we get intermediate values of the expression.)







      share|cite|improve this answer























        up vote
        0
        down vote










        up vote
        0
        down vote









        I tried to give a simple proof by reducing to simpler and simpler cases, all the time doing "nothing", and the final rephrasing is simple, too.




        • First of all, let us note that the (LHS of the) given inequality is invariated by permutations of the letters / variables $a,b,c$. So we may and do assume $ale ble c$.


        • Let $s,t>0$ be then such that $b=a+s$, $c=b+t=a+s+t$.


        • Replacing the initial $a,b,c$ with $-c,-b,-a$, we may and do assume $bge 0$.


        • The given inequality can now be rewritten equivalently:
          $$
          S=
          frac{|a|}t +
          frac{|a+s|}{s+t} +
          frac{|a+s+t|}s
          ge 2 .
          $$


        • If $a$ is $ge0$, then all three numbers $a,b,c$ are $ge 0$, and we can write
          $$
          begin{aligned}
          S
          &= frac{|a|}t +
          frac{|a+s|}{s+t} +
          frac{|a+s+t|}s
          \
          &ge
          frac{0}t +
          frac{s}{s+t} +
          frac{s+t}s
          %ge 2sqrt{
          %frac{s}{s+t} cdot
          %frac{s+t}s}
          ge2 ,qquad(age 0)
          end{aligned}
          $$

          and the last $ge$ (AM-GM) is moreover $>0$ (because $tne 0$). The case with $a < b < c=a+s+tle 0$ is similar.


        • So the single interesting case is the one with $a<0le b<c$ . We rewrite the inequality in the form:
          $$
          frac{|b-s|}t+frac b{s+t}+frac{b+t}sge 2 .
          $$

          Here, $b$ is allowed to vary between $0$ and $s$, and the first modulus is $|a|=-a=-(b-s)=s-b$, so we have to show:
          $$
          frac{s-b}t+frac b{s+t}+frac{b+t}sge 2 .
          $$

          But this is a linear (ergo convex) function in $b$, so the inequality has to be tested only for the possible extremities, $b=0$ and $b=s$. For $b=0$ we obtain $frac st+frac tsge 2$, true, with equality for $s=t$. For $b=s$ we get $frac s{s+t}+frac {s+t}sge 2$, an inequality of the same shape, but we cannot get equality. (For intermediate values of $b$ we get intermediate values of the expression.)







        share|cite|improve this answer












        I tried to give a simple proof by reducing to simpler and simpler cases, all the time doing "nothing", and the final rephrasing is simple, too.




        • First of all, let us note that the (LHS of the) given inequality is invariated by permutations of the letters / variables $a,b,c$. So we may and do assume $ale ble c$.


        • Let $s,t>0$ be then such that $b=a+s$, $c=b+t=a+s+t$.


        • Replacing the initial $a,b,c$ with $-c,-b,-a$, we may and do assume $bge 0$.


        • The given inequality can now be rewritten equivalently:
          $$
          S=
          frac{|a|}t +
          frac{|a+s|}{s+t} +
          frac{|a+s+t|}s
          ge 2 .
          $$


        • If $a$ is $ge0$, then all three numbers $a,b,c$ are $ge 0$, and we can write
          $$
          begin{aligned}
          S
          &= frac{|a|}t +
          frac{|a+s|}{s+t} +
          frac{|a+s+t|}s
          \
          &ge
          frac{0}t +
          frac{s}{s+t} +
          frac{s+t}s
          %ge 2sqrt{
          %frac{s}{s+t} cdot
          %frac{s+t}s}
          ge2 ,qquad(age 0)
          end{aligned}
          $$

          and the last $ge$ (AM-GM) is moreover $>0$ (because $tne 0$). The case with $a < b < c=a+s+tle 0$ is similar.


        • So the single interesting case is the one with $a<0le b<c$ . We rewrite the inequality in the form:
          $$
          frac{|b-s|}t+frac b{s+t}+frac{b+t}sge 2 .
          $$

          Here, $b$ is allowed to vary between $0$ and $s$, and the first modulus is $|a|=-a=-(b-s)=s-b$, so we have to show:
          $$
          frac{s-b}t+frac b{s+t}+frac{b+t}sge 2 .
          $$

          But this is a linear (ergo convex) function in $b$, so the inequality has to be tested only for the possible extremities, $b=0$ and $b=s$. For $b=0$ we obtain $frac st+frac tsge 2$, true, with equality for $s=t$. For $b=s$ we get $frac s{s+t}+frac {s+t}sge 2$, an inequality of the same shape, but we cannot get equality. (For intermediate values of $b$ we get intermediate values of the expression.)








        share|cite|improve this answer












        share|cite|improve this answer



        share|cite|improve this answer










        answered 10 hours ago









        dan_fulea

        5,8201312




        5,8201312






























             

            draft saved


            draft discarded



















































             


            draft saved


            draft discarded














            StackExchange.ready(
            function () {
            StackExchange.openid.initPostLogin('.new-post-login', 'https%3a%2f%2fmath.stackexchange.com%2fquestions%2f3005207%2ffracab-c-fracbc-a-fraccb-a-geq-2%23new-answer', 'question_page');
            }
            );

            Post as a guest















            Required, but never shown





















































            Required, but never shown














            Required, but never shown












            Required, but never shown







            Required, but never shown

































            Required, but never shown














            Required, but never shown












            Required, but never shown







            Required, but never shown







            Popular posts from this blog

            Can a sorcerer learn a 5th-level spell early by creating spell slots using the Font of Magic feature?

            Does disintegrating a polymorphed enemy still kill it after the 2018 errata?

            A Topological Invariant for $pi_3(U(n))$